You are on page 1of 58

PRACTICAL ACCOUNTING 2

THEORY & PRACTICE


COST ACCOUNTING
ABC & BACKFLUSH COSTING
QUIZZER
ABC & Backflush

Backflush Costing System and Activity-Based Costing


Backflush Costing System
I. Introduction
Backflush costing is often used by companies that have adopted a just-in-time (JIT) regarding
inventory control. These companies regard carrying inventory as a nonvalueadded activity.
Hence, they attempt to minimize inventory by making components available just-in-time to be
used in the production process.

II. Definition
Backflush costing or backflushing is a short-cut approach to accounting for the flow of
manufacturing operations. It delays recording journal entires until the goods moved through
the production process.

III. Nature of Backflush Costing System


Backflush costing complements JIT because it simplifies the costing of products. A traditional
costing system tracks costs as they are incurred, but backflush costing delays recording of
some cost information. It treats the detailed recording of inventory data as a non-valueadded
activity. -
The following are the features of Backflush Costing:

1. Work-in process is usually eliminated.


2. Journal entires to inventory accounts may be delayed until the time of product
completion.
3. Standard costs are used to assign costs to units when journal entries are made, i.e. to
flush costs backward to the point at which inventories remain.

Activity-Based Costing
I. Introduction
Activity-based costing (ABC) has been popularized because of the rapid increase in the
automation of manufacturing process, which has led to a significant increase in the
incurrence of indirect costs and a consequent need for more accurate cost allocation.

Under the activity-based costing, as the name implies, costs are accumulated by activity
rather than by department or function for purposes of product costing.

II. Definition
ABC Costing is one means of refining a cost system to avoid what has been called

ABC & Backflush - Lecture Page 1


COST ACCOUNTING

peanut-butter costing. Inaccurately averaging or spreading costs like peanut-butter over


products that use different amounts of resources results in a product-cost-cross-
subsidization.
Product-cost cross-subsidization describes the condition in which the miscasting of one
product causes the miscasting of other products.
In the Accounting Glossary of the Statements of Management Accounting. ABC was
defined as a system that:

1. Identifies the causal relationship between the incurrence of cost and activities
2. Determines the underlying driver of the activities
3. Established cost pools related to individual drivers
4. Develops costing rates
5. Applies cost to product on the basis of resources consumed (drivers)
III. Nature of Activity-Based Costing
In this costing system, cost drivers are identified and used as a basis for cost allocation.
Cost drivers are the factors that will contribute to increasing or decreasing costs. In
addition, under the activity-based costing approach, those activities that do not add
value to the product are reduced to the extent possible.

ABC determines the value-adding activities associated with the incurrence of costs and
then accumulates a cost pool for each activity using the appropriate activity base (cost
driver). Cost pools are then assigned to cost objects. ABC may be employed with job-
order or process-costing methods.
An Activity-Based Costing is a two stage allocation process traced to departments.
First, costs are trace to activities, and then to products.

ABC & Backflush - Lecture Page 2


ABC & Backflush

MCQ - THEORY
1
. An accounting system that collects financial and operating data o the basis of the underlying
nature and extent of the cost drivers is
a. Activity-based costing. c. Delivery cycle time costing.
b. Target costing. d. Variable costing. Punzalan 2014

2
. Multiple or departmental manufacturing overhead rates are considered preferable to a single
or plant-wide overhead when
a. Manufacturing is limited to a single product flowing through identical departments in a
fixed sequence. Punzalan 2014
b. Various products are manufactured that do not pass through the same departments or
use the same manufacturing techniques.
c. Individual cost drivers cannot accurately be determined with respect to cause and effect
relationships.
d. The single or plant-wide rate is related to several identified cost drivers.

3
. Cost drivers are
a. Activities that causes costs to increase as the activity increases.
b. Accounting techniques used to control costs.
c. Accounting measurements used to evaluate whether performance is proceeding
according to plan.
d. A mechanical basis, such as machine hours, computer time, size of equipment, or
square footage of factory, used to assign costs to activities. Punzalan 2014

4
. A base used to allocate the cost of a resource to the different activities using that resource is
called.
a. Resource Driver c. Final cost driver
b. Activity Driver d. Driver Punzalan 2014

5
. A base used to allocate the cost of products, customer, or others final cost objects is called
a. Resource Driver c. Final cost driver
b. Activity Driver d. Driver Punzalan 2014

6
. Activity-based management (ABM) is Punzalan 2014
a. A costing system in which multiple overhead cost pools are allocated using bases that
include one or more non-volume related factors.
b. A base used to allocate the cost of a resource to the different activities using it.

ABC & Backflush – MCQ – Theory Page 3


COST ACCOUNTING

c. The use of information obtained from ABC to make improvements in the firm
d. A base used to allocate the cost of an activity to products and customers.
7
. What is the normal effect on the numbers of cost pools and cost assignment bases when an
activity-based cost system replaces a traditional cost system.
Cost pools Cost assignment bases
a. No Effect No Effect
b. Increase No Effect
c. No Effect Increase
d. Increase Increase Punzalan 2014

8
. Activities, their drivers, and their costs may be classified as unit level, product level, batch
level, and plant level. If activity-based costing information is prepared for internal purposes,
which costs are most likely to be treated as period costs?
a. Unit level. c. Product level.
b. Batch level. d. Plant level. Punzalan 2014

9
. In an activity-based costing system, cost reduction is accomplished by identifying and
eliminating
All Cost Nonvalue-
Drivers adding Activites
a. No No
b. Yes Yes
c. No Yes
d. Yes No Punzalan 2014

10
. Nile Co.'s cost assignment and product costing procedures follow activity-based costing
principles. Activities have been identified and classified as being either value-adding or non
value-adding as to each product. Which of the following activities, used in Nile's production
process, is non value-adding? Punzalan 2014
a. Design engineering activity. c. Drill press activity.
b. Heat treatment activity. d. Raw materials storage activity.
11
. One of the requirements for a JIT system to be successful is
a. Cyclical production.
b. Adequate inventory stock.
c. Coupling it with job order costing.
d. High quality and balanced work loads. Punzalan 2014

ABC & Backflush – MCQ – Theory Page 4


ABC & Backflush
12
. All of the following are terms used to describe the JIT effort to reduce inventories of work in
process and raw materials, except
a. Backflush production c. Lean production
b. Stockless production d. ZIP production Punzalan 2014
13
. In backflush costing, if the conversion cost in the Raw and In Process was P500 on July 1
and P1,000 on July 31, the account to be credited at the end of July for the P500 increase
would be
a. Raw and In Process c. Raw Materials
b. Finished Goods d. Cost of Goods Sold Punzalan 2014
14
. Cost allocation is the process of assigning indirect costs to a cost object. The indirect costs
are grouped in cost pools and then allocated by a common allocation base to the cost object.
The base that is employed to allocate a homopeneous cost pool should
a. Have a cause-and-effect relationship with the cost items in the cost pool.
b. Assign the costs in the pool uniformly to cost objects even if the cost objects use
resources in a non-uniform way.
c. Be a nonfinancial measure (e.g., number of setups) because a nonfinancial measure is
more objective.
d. Have a high correlation with the cost items in the cost pool as the sole criterion for
selection. Dayag 2013
15
. Multiple or departmental overhead rates are considered preferable to a single or plant-wide
overhead rate when
a. Manufacturing is limited to a single product flowing through identical departments in a
fixed sequence.
b. Various products are manufactured that do not pass through the same departments or
use the same manufacturing techniques.
c. Individual cost drivers cannot accurately be determined with respect to cause-and-effect
relationships. Dayag 2013
d. The single or plant-wide rate is related to several identified cost drivers.
16
. An accounting system that collects financial and operating data on the basis of the underlying
nature and extent of the cost drivers is
a. Activity-based costing.
b. Target costing.
c. Cycle-time costing.
d. Variable costing. Dayag 2013
17
. Cost drivers are

ABC & Backflush – MCQ – Theory Page 5


COST ACCOUNTING

a. Activities that cause costs to increase as the activity increases.


b. Accounting techniques used to control costs.
c. Accounting measurements used to evaluate whether performance is proceeding
according to plan.
d. A mechanical basis, such as machine hours, computer time, size of equipment, or
square footage of factory, used to assign costs to activities. Dayag 2013
18
. Which of the following statements about activity-based costing is not true?
a. Activity-based costing is useful for allocating marketing and distribution costs.
b. Activity-based costing is more likely to result in major differences from traditional costing
systems if the firm manufactures only one product rather than multiple products.
c. In activity-based costing, cost drivers are what cause costs to be incurred.
d. Activity-based costing differs from traditional costing systems in that products are not
cross-subsidized. Dayag 2013

19
. Because of changes that are occurring in the basic operations of many firms, all of the
following represent trends in the way indirect costs are allocated except.
a. Treating direct labor as an indirect manufacturing cost in an automated factory.
b. Using throughput time as an application base to increase awareness of the costs
associated with lengthened throughput time.
c. Preferring plant-wide application rates that are applied to machine hours rather than
incurring the cost of detailed allocations.
d. Using several machine cost pools to measure product costs on the basis of time in a
machine center. Dayag 2013

20
. What is the normal effect on the numbers of cost pools and allocation bases when an activity-
based cost (ABC) system replaces a traditional cost system?
Cosf Pools Allocation Bases
a. No effect No effect
b. Increase No effect
c. No effect Increase
d. Increase Increase Dayag 2013
21
. In an activity-based costing (ABC) system, cost reduction is accomplished by identifying and
eliminating
All Cost Drivers Nonvalue-Adding Activities
a. No No
b. Yes Yes

ABC & Backflush – MCQ – Theory Page 6


ABC & Backflush

c. No Yes
d. Yes No Dayag 2013
22
. Which of the following would be a reasonable basis for allocating the materials handling costs
to the units produced in an activity-based costing system?
a. Number of production runs per year.
b. Number of components per completed unit.
c. Amount of time required to produce one unit.
d. Amount of overhead applied to each completed unit. Dayag 2013
23
. Chang Co.'s cost allocation and product costing procedures follow activity-based costing
principles. Activities have been identified and classified as being either value-adding or
nonvalue-adding as to each product. Which of the following activities, used in Chang's
production process, is nonvalue-adding?
a. Design engineering activity.
b. Heat treatment activity.
c. Drill press activity.
d. Raw materials storage activity. Dayag 2013

24
. A streamlined accounting system:
a. Standard costing c. Process costing
b. Job Order costing d. Backflush costing Dayag 2013

ABC & Backflush – MCQ – Theory Page 7


COST ACCOUNTING

ACTIVITY-BASED COSTING
Consumption Ratio
Questions 1 & 2 are based on the following: Dayag 2013
25
. Principal, Inc. produces three products. Production and cost information is as follows:
Model A Model B Model C
Units produced 2,000 6,000 12,000
Direct labor hours 4,000 2,000 4,000
Number of setups 100 150 250
The consumption ratios for number of setups would be:
A B C A B C
a. 40% 20% 40% c. 10% 30% 60%
b. 20% 30% 50% d. 5% 22% 73%

26
. Using the same information in No. 40, compute the consumption ratios based on units
produced would be:
A B C A B C
a. 40% 20% 40% c. 10% 30% 60%
b. 20% 30% 50% d. 5% 22% 73%

Activity Rates
Comprehensive
27
. Product 53 uses 200 hours of direct labor and has 2,000 machine steps. Kate Ong, the cost
accountant, has been considering using either direct labor hours or machine steps as the cost
driver. The ratio of overhead cost to direct labor hours is P60. The assignment of overhead
cost to Product 53 using direct labor hours would result in a higher charge by P4,000 than if
machine steps were used as the cost driver.
Determine the ratio of overhead cost to machine steps.
a. P6 c. P60
b. 2 d. 4 Dayag 2013

Applied Overhead
Labor-Related Overhead Cost
28
. Cebu Enterprises is a Philippine exporter of souvenir items manufactured in the capital city of
Jerusalem. The following overhead cost data have been accumulated:
Amount
Activity Center Cosf Driver of Activity Center Costs
Materials Handling Grams handled 100,000 grams P 50,000

ABC & Backflush – MCQ – Problems Page 8


ABC & Backflush

Painting Units painted 50,000 units 200,000


Assembly Labor hours 4,000 hours 120,000
Job 1234 contains 3,000 units. It weighs 10,000 grams and uses 300 hours of labor.
Compute the total overhead costs that should be assigned to Job 1234.
a. P31,955 c. P26,000
b. 27,750 d. 32,000 Dayag 2013

Comprehensive
Questions 1 & 2 are based on the following: Dayag 2013
29
. UST Community Hospital has found itself under increasing pressure to be accountable for the
charges it assesses its patients. Its current pricing system is ad hoc, based on pricing norms
for the geographical area, and it only explicitly considers direct costs for surgery, medication,
and other treatments. UST's controller has suggested that the hospital try to improve its
pricing policies by seeking a tighter relationship between costs and pricing. This approach
would make prices for services less arbitrary. As a first step, the controller has determined
that most costs can be assigned to one of three cost pools. The three cost pools follow along
with the estimated amounts and activity drivers.

Activity Center Amount Activity Driver


Quantity
Professional salaries P900,000 Professional hours 30,000 hours
Building costs 450,000 Square feet used 15,000 square feet
Risk management 320,000 Patients served 1,000 patients

The hospital provides service in three broad categories. The services are listed below with
their volume measures for the activity centers.
Service Professional Hours Square Feet Number of Patients
Surgery 6,000 1,200 200
Housing patients 20,000 12,000 500
Outpatient care 4,000 1,800 300

Compute the allocation rates for each activity center:


Professional Building Risk
Salaries Costs Management
a. P122/hr. P14/sq. ft. P52 per patient
b. 22/hr. 22/sq. ft. 22 per patient

ABC & Backflush – MCQ – Problems Page 9


COST ACCOUNTING

c. 30/hr. 30/sq. ft. 320 per patient


d. 150/hr. 37.50/sq. ft. 1,067 per patient

30
. Using the same information in No. 29, determine the allocated activity center costs to the
three services provided by the hospital:
Surgery Housing Outpatient Care
a. P759,200 P2,634,000 P528,800
b. 162,800 715,000 134,200
c. 2,828,400 3,983,500 987,600
d. 280,000 1,120,000 270,000

31
. Delta Machine Tool Incorporated produces a varied product line without the use of direct
labor. An extensive setup procedure is required. Because no single base for a predetermined
overhead rate will provide Delta with reliable product cost information, overhead is classified
into two cost pools and two predetermined overhead rates are used. For 2013, it is estimated
that total overhead costs will consist of P525,000 of overhead related to setups and P900,000
of overhead related to ma¬ chine usage. Total machine usage is expected to be 3,600 hours
for the year, and the total number of setups is expected to be 300. Job RST required parts
and materials costing P56,000, 70 hours of machine time, and four setups. What is the cost of
Job RST?
a. P80,500 c. P83,050
b. P78,500 d. P79,500 Guerrero 2013

Unit Cost
Questions 1 & 2 are based on the following: Dayag 2013
32
. Katherine Inc. of Cebu Ltd. has decided to institute a pilot activity-based costing project in its
five-person purchasing department. Annual departmental costs are P473,500. Because
finding the best supplier takes the majority of effort in the department, most of the costs are
allocated to this area.
Number of Total
Activity Allocation Measure People Cost
Find best suppliers Number of telephone calls 3 P300,000
Issue purchase orders Number of purchase orders 1 100,000
Review receiving reports Number of receiving reports 1 73,500
During the year, the purchasing department made 150,000 telephone calls, issued
10,000 purchase orders, and reviewed 7,000 receiving reports. Many purchase orders

ABC & Backflush – MCQ – Problems Page 10


ABC & Backflush

are received in a single shipment.


One product manufactured by Cebu Ltd. required the following purchasing department
activities: 125 telephone calls, 60 purchase orders, and 15 receipts.
What amount of purchasing department cost should be assigned to the product?
a. P1,007.50 c. P921.48
b. 4,500.00 d. 327.55
33
. Using the same information in No. 27, what is the purchasing department cost per unit would
be if 200 units of the product are manufactured during the year?
a. P2.00 c. P10.00
b. 5.04 d. 10.50
34
. Believing that its traditional cost system may be providing misleading information, BMW
company is considering an activity based costing approach. It now employs a full cost system
and has been applying its manufacturing overhead on the basis of machine hours. The
company plans on using 50,000 direct labor hours and 30,000 machine hours in the coming
year. The following data show the manufacturing overhead that is budgeted.
Activity Cost Driver Budgeted Activity Budgeted Cost
Material Handling No. of parts handled 6,000,000 P720,000
Setup costs No. of setups 750 315,000
Machining costs Machine hours 30,000 540,000
Quality control No. of batches 500 225,000
Cost, sales, and production data for one of the company's product for the coming year are as
follows:
Prime Costs:
Direct material cost per unit P4.40
Direct labor cost per unit, .05 direct labor
hour@P 15 per hour 0.75
Sales and production data:
Expected sales 20,000units
Batch size 5,000units
Setups 2per batch
Total parts per finished unit 5parts
Machine hours required 80machine hours per batch
If the company employs an activity-based costing system, the cost per unit for the product
described for the coming year will be:
a. P6.00 c. P6.21
b. P6.08 d. P6.30 Guerrero 2013

ABC & Backflush – MCQ – Problems Page 11


COST ACCOUNTING
35
. Uratex Company manufactures a variety of classroom chairs. Its job-costing system uses an
activity-based approach. There are two direct-cost categories (direct materials and direct
labor) and three indirect cost pools. The cost pools represent three activity areas at the plant.
Manufacturing Budgeted Cost Driver used Cost-allocation
Activity Area Cost for 2013 as Allocation Base Rate
Materials Handling P 200,000 Parts P 0.25
Cutting 2,000,000 Parts 2.50
Assembly 2,000,000 Direct labor hours 25,000
Two styles of chairs were produced in March, the high school chair, and the college chair.
Their quantities, direct material costs, and other data for March 2013
are as follows:
Direct Direct
Units Materials Number Manufacturing
Produced Costs of Parts Labor hours
High school chair 5,000 P600,000 100,000 7,500
College chair 100 25,000 3,500 500
The direct labor rate is P20 per hour. Assume no beginning or ending inventory. What are the
unit cost of the high school chair and the college chair?
a. P240.50 and P571.75 respectively
b. P242.50 and P570.25 respectively
c. P252.50 and P571.25 respectively
d. P242.50 and P571.25 respectively Guerrero 2013

Comprehensive
Activity Rate, Unit Cost
Questions 1 & 2 are based on the following: Dayag 2013
36
. Ongchan Metal Fabricators has a diverse product line and a complex cost structure, with
some jobs requiring much labor and little machine use and others requiring the opposite mix.
Because no single base for a predetermined overhead rate will provide Ongchan Metal
management with reliable product cost information, overhead is classified into two cost pools
and two predetermined overhead rates are used. For 2001, it is estimated that total overhead
costs will consist of P200,000 of overhead related to the usage of direct labor hours and
P300,000 of overhead related to machine usage. Total machine usage is expected to be
4,000 hours for the year, and total direct labor hours are expected to be 16,000.
Job 345 required P2,000 of direct material, 30 hours of labor at P10 per hour, and 10
hours of machine time.
Determine the dual predetermined overhead rates:

ABC & Backflush – MCQ – Problems Page 12


ABC & Backflush

Per Direct Labor Hour Per Machine Hour


a. P 3.75 P75.00
b. 375.00 7.50
c. 12.50 12.50
d. 12.50 75.00
37
. Using the same information in No. 1, compute the cost of Job 345:
a. P2,300.00 c. P3,425.00
b. 3,162.50 d. 5,800.00

Questions 1 & 2 are based on the following: Dayag 2013


38
. Kate Machine Tool Incorporated produces a varied product line without the use of direct labor.
An extensive setup procedure is required. Because no single base for a predetermined
overhead rate will provide Kate with reliable product cost information, overhead is classified
into two cost pools and two predetermined overhead rates are used. For 2011, it is estimated
that total overhead costs will consist of P525,000 of overhead related to setups and P900,000
of overhead related to machine usage. Total machine usage is expected to be 3,600 hours for
the year, and the total number of setups is expected to be 300.
Job 403 required parts and materials costing P56,000, 70 hours of machine time, and
four setups.

Determine the dual predetermined overhead rates:


Per Machine Hour Per Setup
a. P250.00 P1,750.00
b. 1,750.00 250.00
c. 145.83 3,000.00
d. 365.38 365.38

39
. Using the same information in No. 1, compute the cost of Job 403
a. P78,208 c. P 83,038
b. 80,500 d. 179,500

Applied Overhead, Unit Cost & Unit Price


Questions 1 thru 3 are based on the following: Dayag 2013
40
. A company has identified the following overhead costs and cost drivers for the coming year:

Overhead Item Cost Driver Budgeted Cost Budgeted

ABC & Backflush – MCQ – Problems Page 13


COST ACCOUNTING

Activity Level
Machine setup Number of setups P 20,000 200
Inspection Number of inspections P130,000 6,500
Material Handling Number of material P 80,000 8,000
moves
Engineering Engineering hours P 50,000 1,000
P280,000
The following information was collected on three jobs that were completed during the year:

Job 101 Job 102 Job 103


Budgeted direct labor Direct materials P5,000 P12,000 P8,000
cost was P100,000 Direct labor P2,000 P2,000 P4,000
and budgeted direct Units completed 100 50 200
material cost was Number of setups 1 2 4
P280,000. Number of 20 10 30
inspections
If the company
Number of material 30 10 50
uses activity-
based costing, moves
how much Engineering hours 10 50 10
overhead cost should be allocated to Job 101 ?
a. P1,300 c. P5,000
b. 2,000 d. 5,600

41
. Using the same information in No. 1, compute the cost of each unit of Job 102 using Activity-
Based Costing:
a. P340 c. P440
b. 392 d. 520

42
. Using the same information in No. 1, assuming the company prices its products at 140% of
cost and the company uses Activity-Based Costing, the price of each unit of Job 103 would
be:
a. P 98 c. P116
b. 100 d. 140

ABC & Backflush – MCQ – Problems Page 14


ABC & Backflush

Activity Rate & Applied Overhead


43
. Tomas Company manufactures two types of medical syringes: low-unit and med-unit. The
overhead activities, costs, and related data are as follows:
Low-Unit Med-Unit Activity Center
Costs
Receiving orders 100 150 P7,500
Machine hours 12,000 13,000 125,000
Setups 45 20 9,750
Shipping orders 200 400 30,000
Assume the total costs of all activity centers are allocated on the basis of machine hours.
Calculate the overall rate and allocate overhead costs to the two products using that rate.
Low-Unit Med-Unit Low-Unit Med-Unit
a. P89,570 P82,680 c. P82,680 P82,680
b. 82,680 89,570 d. 89,570 89,570 Dayag 2013

Questions 1 thru 3 are based on the following: Punzalan 2014


New Wave Co. produces two products, Product Side A and Product Side B. Units produced for the
period totaled 13,625 for Side A and 11,100 for Side B. Total manufacturing overhead cost are
P550,000. The company uses the activity-based costing in allocating its manufacturing overhead
cost. The following additional information were gathered:
Product
Activity Cost Driver A B Total
Setups No. of setups 100 400 100,000
Machining Machine hours 3,000 2,000 400,000
Inspections No. of inspections 100 300 50,000
44
. What would be the overhead rate for each activity?
Setups Machining Inspections
a. 200.00 80.00 125.00
b. 100.00 40.00 62.50
c. 183.33 183.33 183.00
d. 25.00 45.00 20.00

45
. How would the total overhead cost of P550,000 be allocated between
Products Side A and Side B?
Side A Side B
a. 640,000 540,000
b. 586,656 494,991

ABC & Backflush – MCQ – Problems Page 15


COST ACCOUNTING

c. 275,000 275,000
d. 272,500 277,500

46
. What would be the overhead cost per unit for Products Side A and Side B?
Side A Side B
a. P46.97 P48.65
b. 43.06 44.59
c. 20.18 24.77
d. 20.00 25.00

ABC & TRADITIONAL COSTING


Overhead Cost per Unit
Setup Cost
47
. Product BDE uses 200 hours of direct labor hours and has 2,000 machine steps. The
company has been considering using either direct labor hours or machine steps as the cost
driver. The ratio of overhead cost to direct labor hours is P60. The assignment of overhead
cost to Product BDE using the direct labor hours would result in a higher charge by P4,000
than if machine steps were used as the cost driver. What is the ratio of overhead cost to
machine steps?
a. 1.00 per step c. 4.00 per step
b. 2.00 per step d. 6.00 per step Punzalan 2014

48
. Yokomo Inc. accumulated the following cost information for its two products, A and B.

Product A Product B
Units produced 2,000 1,000
Total direct labor hours 5,000 20,000
Set-up cost per batch PI,000 P2,000
Batch size 100 50
Total set-up cost incurred P20,000 P40,000
Direct labor hour per unit 2 1
A traditional costing system would allocate setup costs on the basis of direct labor hours. An
ABC system would trace costs by spreading the cost per batch over the units in a batch. What
is the setup cost per unit of Product A under each costing system.
Traditional Costing ABC
A. P 4.80 P 10
B. P 2.40 P 10

ABC & Backflush – MCQ – Problems Page 16


ABC & Backflush

C. P40.00 P200
D. P4.80 P 20 Guerrero 2013

Material Handling Cost


Questions 1 & 2 are based on the following: Dayag 2013
49
. Elvie manufactures two versions of a product. Production and cost information show the
following:
Model X Model Y
Units produced 100 200
Material moves 10 40
Direct labor hours per unit 1 2
Materials handling costs total PI00,000. Under ABC, the materials handling costs allocated to
each unit of Model would be:
Model X Model Y Model X Model Y
a. P400 P200 c. P143 P429
b. 200 400 d. 429 143

50
. Using the same information in No. 1, except that direct labor hours (traditional / conventional)
were used to allocate overhead costs. The material handling costs allocated to each unit of
Model would be (assuming that 2 DLH per unit for Model Y):

Model X Model Y Model X Model Y


a. P400 P200 c. P143 P429
b. 200 400 d. 429 143

Questions 1 & 2 are based on the following: Dayag 2013


51
. Zeta Company is preparing its annual profit plan. As part of its analysis of the profitability of
individual products, the controller estimates the amount of overhead that should be allocated
to the individual product lines from the information given as follows:

Wall Specialty
Mirrors Windows
Units produced 25 25
Material moves per product line 5 15
Direct labor hours per unit 200 200
Budgeted materials handling costs. P50,000

ABC & Backflush – MCQ – Problems Page 17


COST ACCOUNTING

Under a costing system that allocates overhead on the basis of direct labor hours (traditional),
the materials handling costs allocated to one unit of wall mirrors would be
a. P1,000 c. P2,000
b. 500 d. 5,000

52
. Using the same information in No. 1, the materials handling costs allocated to one unit of wall
mirrors under Activity-Based Costing would be:
a. P1,000 c. P1,500
b. 500 d. 2,500

Numbers 1 and 2 are based on the following data: Guerrero 2013


Omega company is preparing its annual profit plan. As part of its analysis of the profitability of
individual products, the controller estimates the amount of overhead that should be allocated to the
individual product lines from the information given as follows:

Wall Specialty
Mirrors Windows
Units produced 25 25
Material moves per product line 5 15
Direct labor hours per unit 200 200
Budgeted materials handling costs P50,000

53
. Under a costing system that allocates overhead on the basis of direct labor hours (traditional),
the materials handling costs allocated to one unit of wall mirrors Would be
a. P1,000 c. P2,000
b. P500 d. P5,000
54
. Under activity based costing (ABC), what is the materials handling costs allocated to one unit
of wall mirrors?
a. P1,000 c. P1,500
b. P500 d. P2,500
Comprehensive
Questions 1 thru 4 are based on the following: Dayag 2013
55
. Ambrose Company uses three products with the following production and cost information:

Model S Model M Model L


Units produced 2,000 6,000 12,000

ABC & Backflush – MCQ – Problems Page 18


ABC & Backflush

Direct labor hours (total) 4,000 2,000 4,000


Number of setups 100 150 250
Number of shipments 200 225 275
Engineering change orders 15 10 5

Overhead costs include setups of P45,000; shipping cost of P70,000; and engineering
costs of P90,000. What would be the per unit overhead cost for each model if direct
labor hours (traditional/conventional costing) were the allocation base?
Model S Model M Model S Model M.
a. P10.25 P10.25 c. P20.50 P20.50
b. 41.00 20.50 d. 41.00 6.83

56
. Using the same information in No. 1, what would be the per unit overhead cost for Model S, if
ABC costing were used?
a. P10.25 c. P41.00
b. 37.00 d. 20.50

57
. Using the same information in No. 1, what would be the per unit overhead cost for Model M, if
ABC costing were used?
a. P11.00 c. P61.50
b. 33.00 d. 41.00

58
. Using the same information in No. 1, what would be the per unit overhead cost for Model L, if
ABC were used?
a. P 5.42 c. P122.64
b. 16.25 d. 41.00

59
. In manufacturing roller blades, Wesley Co.'s plant used 400 direct labor hours, 500 machine
hours, and 20 set ups. The following overhead costs were taken from the factory accounts:
Overhead Expenses Volume of Activities
Machine center P 120,000 20,000 machine hours
Set up center 40,000 100 set ups
Total P 160,000 4,000 direct labor hours
The plant was using a factory-wide overhead rate based on direct labor hours. A new ABC
system will use machine hours in the Machining Department and number of set ups in the Set
up Department as cost drivers. By what amount would overhead costs assigned to roller
blades differ between the prior system and the ABC system?

ABC & Backflush – MCQ – Problems Page 19


COST ACCOUNTING

a. 7,800 c. 5,000
b. 7,000 d. 4,800 Punzalan 2014

Net Income (Loss)


Questions 1 & 2 are based on the following: Dayag 2013
60
. Sony Ericsson has two major components with the following information:
Indoor Outdoor
Furniture Furniture Total
Annual revenue P300,000 P600,000 P900,000
Material costs P 40,000 P 60,000 P100,000
Labor costs P 50,000 P 75,000 P125,000
Materials hours 80,000 20,000
Number of batches 100 100
The business also has overhead costs as follows:
Cost Pool Cost in Pool Cost driver
Maintenance P200,000 Machine hours
Setups 175,000 Number of batches
Administrative 125,000 Labor costs
P500,000
Determine the income (loss) of each segment if the overhead costs will be allocated to
segment based on labor costs (traditional):
Indoor Outdoor Indoor Outdoor
a. P87,500 P(262,500) c. P10,000 P165,000
b. (87,500) 262,500 d. (10,000) (165,000)

61
. Using the same information in No. 1, determine the income (loss) of each segment if the
overhead costs will be allocated to segment under ABC costing:
Indoor Outdoor Indoor Outdoor
a. P87,500 P(262,500) c. P10,000 P165,000
b. 87,500 262,500 d. (10,000) (165,000)

Total Production Cost


62
. New-Rage cosmetics has used a traditional cost accounting system to apply quality control
costs uniformly to all products at a rate of 14.5% of direct labor cost. Monthly direct labor cost
for Satin Sheen makeup is P27,500. In an attempt to distribute quality control costs more
equitably, New-Rage is considering activity-based costing. The monthly data shown in the
chart below have been gathered for Satin Sheen makeup.

ABC & Backflush – MCQ – Problems Page 20


ABC & Backflush

Quantity for
Activity Cost Driver Cost Rates Satin Sheen
Income
material
inspection Type of material P11.50 per type 12 types
In-process
Inspection Number of units P0.14 per unit 17,500 units
Product
Certification Per order P77 per order 25 orders

The monthly quality control cost assigned to Satin Sheen makeup using activity-based
costing is
a. P88.64 per order.
b. P525.50 lower than the cost using the traditional system.
c. P8,500.50
d. P525.50 higher than the cost using the traditional system. Dayag 2013

Questions 1 & 2 are based on the following: Dayag 2013


63
. Believing that its traditional cost system may be providing misleading information, an
organization is considering an activity-based costing (ABC) approach. It now employs a full-
cost system and has been applying its manufacturing overhead on the basis of machine
hours.
The organization plans on using 50,000 direct labor hours and 30,000 machine hours in
the coming year. The following data show the manufacturing overhead that is budgeted.
Budgeted Budgeted
Activity Cost Driver Activity Cost
Materials handling No. of parts handled 6,000,000 P720,000
Setup costs No. of setups 750 315,000
Machining costs Machine hours 30,000 540,000
Quality control No. of batches 500 225,000
Total manufacturing overhead cost: P1,800,000

Cost, sales, and production data for one of the organization's products for the coming year

ABC & Backflush – MCQ – Problems Page 21


COST ACCOUNTING

are as follows:
Prime costs:
Direct material cost per unit P4.40
Direct labor cost per unit
.05 DLH @ P15.00/DLH 75
Total prime cost P5.15

Sales and production data:


Expected sales 20,000 units
Batch size 5,000 units
Setups 2 per batch
Total parts per finished unit 5 parts
Machine hours required 80 MH per batch
If the organization uses the traditional full-cost system, the cost per unit for this product for the
coming year would be
a. P5.39 c. P6.11
b. 5.44 d. 6.95

64
. Using the same information in No. 1, and assuming that the organization employs an Activity-
Based Costing System, the cost per unit of the product described for the coming year would
be:
a. P6.00 c. P6.21
b. 6.08 d. 6.30
65
. Tamiya Corporation has used a traditional costing system to apply quality control costs
uniformly to all products at a rate of 14.5% of direct labor cost. Monthly direct labor cost for its
Product X is P275,000. In an attempt to distribute quality control costs more equitable, Tamiya
is considering activity-based costing (ABC). The June data shown below have been gathered
for Product X.

Activity Cost Driver Cost Rates Quantity


1. Material handling Type of materials P115 per type 12 types
2. Inspection Number of units P1.40 per unit 17,500 units
3. Product certification Per order P770 per order 25 orders

What is the monthly quality control assigned to product X using the ABC?
a. P686.40 per order.
b. P5,255 higher than the traditional costing system.

ABC & Backflush – MCQ – Problems Page 22


ABC & Backflush

c. P8 5,000
d. P5,255 lower than the traditional costing system. Guerrero 2013

66
. The Manila Company manufactures and sells packaging machines. It recently used an
activity-based approach to refine the job costing system at its Bulacan plant. The resulting job
costing system has one direct-cost category (direct materials) and four indirect manufacturing
cost pools. These four indirect cost pools and their allocation bases are:

Indirect Manufacturing Cost-Allocation Budgeted Cost-


Cost Pool Base Allocation Rate
1. Material handling Component parts P8 per part
2. Machining Machine-hours P68 per hour
3. Assembly Assembly-hours P75 per hour
4. Inspection Inspection-hours PI04 per hour

Manila Company recently sold 50 can-packaging machines to Ilocos Company. Each


machine has direct material costs of P3,000 requires 50 component parts, 12 machine-hours,
15 assembly-hours, and 4 inspection hours. Manila Company's previous costing system had
one direct-cost category (direct materials) and one indirect-cost category (manufacturing
overhead allocated at the rate of PI00 per assembly-hour). In comparison to the traditional
costing system used by Manila Company, the total manufacturing cost of the machines sold
under the ABC is:
a. P114,850 higher c. P114,950 higher
b. P141,850 lower d. Equal Guerrero 2013

ABC vs TRADITIONAL COSTING


Cross Subsidy per Unit
67
. Factory Co. makes two products, X and Z. X is being introduced this period, whereas Z has
been in production for 2 years. For the period about to begin, 1,000 units of each product are
to be manufactured. Assume that the only relevant overhead item is the cost of engineering
change orders; that X and Z are expected to require 8 and 2 change orders, respectively; that
X and Z are expected to require 2 and 3 machine hours, respectively; and that the cost of a
change order is P600. If Factory Co.- applies engineering change order costs on the basis of
machine hours, the cross-subsidy per unit arising from this peanut butter costing approach is
a. 1.20 c. 3.60
b. 2.40 d. 4.80 Punzalan 2014

ABC & Backflush – MCQ – Problems Page 23


COST ACCOUNTING

BACKFLUSHING
RIP to Finished Goods
68
. The Pampanga Manufacturing Company uses a raw and in process (RIP) inventory account
and expenses all conversion costs to the cost of goods sold account. At the end of each
month, all inventories are counted, their conversion cost components are estimated, and
inventory account balances are adjusted accordingly. Raw material cost is backflushed from
RIP to Finished Goods. The following information is for the month of April:
Beginning balance of RIP account, including P1,400
of conversion cost P 31,000
Raw materials received on credit 367,000
Ending RIP inventory per physical count, including
P1,800 conversion cost estimate 33,000
Compute the amount to be backflushed from RIP to Finished goods:
a. P365,000 c. P367,000
b. 368,600 d. 365,400 Dayag 2013

69
. The Backflushers Manufacturing Corp. uses a Raw and In Process inventory account and
expenses all conversion costs to the cost of goods sold account. At the end each month, all
inventories are counted, their conversion cost components are estimated and inventory
account balances are adjusted accordingly. Raw material cost is backflushed from RIP to
finished goods. The following information is for the month of May:
Raw and In Process inventory account, May 1, including
P500 of conversion cost P 5,000
Raw materials received during May (50% down, balance in
four installments) 100,000
Raw and In Process inventory account, May 31, including
P650 of estimated conversion cost 5,250
What is the amount to be backflushed from RIP to finished goods?
a. 90,900 c. 104,500
b. 99,900 d. 109,100 Punzalan 2014
70
. Cheeta Co. has materials cost in the June 1 Raw and In Process (RIP) of PI0,000 materials
received during the month of June of P205,000 and materials cost in the June 30 Raw and In
Process (RIP) of P 12,500. What would be the amount to be backflushed from RIP to
Finished Goods at the end of June?
a. 215,000 c. 207,500
b. 202,500 d. 217,500 Punzalan 2014

ABC & Backflush – MCQ – Problems Page 24


ABC & Backflush
71
. The Hudy Manufacturing Company uses a Raw and In Process (RIP) inventory account and
expensed all conversion costs to the cost of goods sold account. At the end of each month,
all inventories are counted, their conversion cost components are estimated, and inventory
account balances are adjusted accordingly. Raw material cost is backflushed from RIP to
finished Goods. The following information is for the month of April:
Beginning balance of RIP account, including PI,400 .
of conversion cost P 31,000
Raw materials received on credit 367,000
Ending RIP inventory per physical count, including
P1,800 conversion cost estimate 33,000
What is the amount of materials used to be backflushed from RIP to finished goods?
a. P365,000 c. P367,000
b. P368,600 d. P365,400 Guerrero 2013

72
.
The Futaba Manufacturing Company uses a Raw and In Process (RIP) inventory account. At
the end of each month, all inventories are counted, their conversion costs components are
estimated, and inventory account balances are adjusted accodingly. Raw materials cost is
backflushed form RIP account to Finished Goods account. The following data is for the month
of August:
Beginning balance of RIP account P38,700
Conversion costs incurred 4,800
Raw materials purchased 680,000
Conversion costs allocated 5,300
Ending balance of RIP account 41,900
The amount of direct materials and conversion costs to be backflushed to finished goods are:
a. P676,800 and P4,800 respectively
b. P680,000 and P4,800 respectively
c. P676,800 and P5,300 respectively
d. P680,000 and P5,300 respectively Guerrero 2013
RIP to Cost of Goods Sold
73
. The HPI Manufacturing Company produces only for customer order and most work is shipped
within thirty-six hours after the receipt of an order. HPI uses a Raw and In Process (RIP)
inventory account and expensed all conversion costs to the cost of goods sold account. Work
is shipped immediately upon completion, so there is no finished goods account. At the end of
each month, inventory is counted, its conversion cost component is estimated, and the RIP
account balance is adjusted accordingly. Raw material cost is backflushed from RIP to Cost
of Goods Sold. The following information is for the month of May:

ABC & Backflush – MCQ – Problems Page 25


COST ACCOUNTING

Beginning balance of RIP account, including P1,300


of conversion cost P12,3 00
Raw materials received on credit 246,000
Ending RIP inventory per physical count, including
P2,100 conversion cost estimate 12,100
What is the amount of raw materials used to be backflushed from RIP to cost of goods sold?
a. P246,000 c. P247,000
b. P246,200 d. P245,000 Guerrero 2013

RIP Account
74
. The Love Company seeks to streamline the costing system at its Manila plant, it will use a
backflush costing system with three trigger points:
Purchase of raw materials
Completion of finished goods
Sale of finished goods
There are no beginning inventories. The following data pertain to April 2013:
Raw materials purchased P880,000
Raw materials used 850,000
Conversion costs incurred 422,000
Conversion allocated to finished goods 400,000
Costs transferred to finished goods 1,250,000
Costs of goods sold 1,190,000
Assume no materials variances. The balance of RIP account at the end of april 2013 is:
a. P 30,000 c. P850,000
b. P880,000 d. P0 Guerrero 2013

RIP Account & Cost of Goods Sold


75
. The Pit Shop Company produces telephones. For June, there were no beginning inventory of
raw materials and no beginning and ending work in process. Pit Shop uses a JIT
manufacturing system and backflush costing with two trigger points for making entries in its
accounting system.
Purchase of raw materials
Sales of finished goods

Pit Shop's standard cost per unit of telephone in June are direct materials, P26; and
conversion costs, PI5. The following data apply to June production:
Raw materials purchased P5,300,000

ABC & Backflush – MCQ – Problems Page 26


ABC & Backflush

Conversion costs incurred 3,080,000


Number of finished units manufactured 200,000
Number of finished units sold 192,000

The balances of Raw and In Process and Cost of Goods Sold accounts at the end of June
are:
a. P308,000 and P7,872,000 respectively
b. P5,300,000 and P7,872,000 respectively
c. P308,000 and P4,992,000 respectively
d. P4,992,000 and P2,880,000 respectively Guerrero 2013

RIP Account & Finished Goods


76
. The Action Corporation manufactures electrical meters. For May, there were no beginning
inventories of raw materials and no beginning and ending work in process. Action uses a JIT
manufacturing system and backflush costing with three trigger points for making entries in the
accounting system:

Purchase of raw materials - debited to Raw and In Process account


Completion of finished goods - debited to Finished Goods account
Sale of finished goods
Action's May standard cost per meter are direct materials, P25; and conversion costs, P20.
The following data apply to May manufacturing:
Raw materials and components purchased P550,000
Conversion costs incurred P440,000
Number of finished units manufactured 21,000
Number of finished units sold 20,000

The balances of Raw and in Process and Finished Goods inventory accounts at the end of
May are:
A. P25,000 and P945,000 respectively
B. P550,000 and P45,000 respectively
C. P25,000 and P45,000 respectively
D. P550,000 and P945,000 respectively Guerrero 2013

Raw Materials Purchased


77
. If Edsa Company has material cost of P10,000 in the June 1 RIP inventory account, and
P12,500 in the June 30 RIP inventory account and the amount of raw materials used

ABC & Backflush – MCQ – Problems Page 27


COST ACCOUNTING

backflushed from RIP inventory account on June 30 is P202,500, what is the amount of raw
materials purchased on credit for the month of June?
a. P205.000 c. P22 5,000
b. P200,000 d. P200.000 Guerrero 2013
Conversion Cost in Units Sold
78
. Basilio Company has a cycle time of 3 days, uses a Raw and In Process (RIP) account, and
charges all conversion costs to Cost of Goods Sold. At the end of each month, all inventories
are counted, their converison costs components are estimated, and inventory account
balances are adjusted. Raw material cost is backflushed from RIP to Finished Goods. The
following information is for June:
Beginning balance of RIP account, including P3,000
of conversion costs P29,250
Beginning balance of finished goods account,
including P10,000 of conversion costs 30,000
Raw materials received on credit 562,500
Direct labor cost, P375,000; Factory overhead applied,
P450,000 825,000
Ending RIP inventory per physical count, including
P4,500 of conversion costs 32,000
Ending finished goods inventory per physcal count,
including P8,750 of convesion costs 26,250
What is the conversion costs of units sold in June?
a. P825,250 c. P840,000
b. P825,000 d. P824,750 Guerrero 2013
Conversion Cost & Finished Goods
Questions 1 & 2 are based on the following: Dayag 2013
79
. Changi Ong, general manager of Casio Corporation's Midwest Division, has provided the
following information for transactions that occurred during March. This division uses a JIT
costing system.
(a) Raw materials were purchased at the cost of P97,000.
(b) All materials purchased were requisitioned for production.
(c) Direct labor costs of P77,000 were incurred.
(d) Actual factory overhead costs amounted to P225,000.
(e) Applied conversion costs totaled P300,000. This included P77,000 of direct labor.
(f) All units were completed.
Compute the March 31 balance in the Conversion Cost:

ABC & Backflush – MCQ – Problems Page 28


ABC & Backflush

a. P2,000 debit c. P25,000 credit


b. 2,000 credit d. 22,000 debit

80
. Using the same information in No. 1, compute the March 31 balance in the Finished goods
account:
a. P397,000 credit c. P320,000 debit
b. 397,000 debit d. 377,000 debit

Comprehensive
Questions 1 & 2 are based on the following: Dayag 2013
81
. The Bulacan Manufacturing Company produces only for customer order and most work is
shipped within thirty-six hours of the receipt of an order. Bulacan uses a raw and in process
(RIP) inventory account and expenses all conversion costs to the cost of goods sold account.
Work is shipped immediately upon completion, so there is no finished goods account. At the
end of each month, inventory is counted, its conversion cost component is estimated, and the
RIP account balance is adjusted accodingly. Raw material cost is backflushed from RIP to
Cost of Goods Sold. The following information is for the month of May:

Beginning balance of RIP account, including P1,300


of conversion cost P12,300
Raw materials received on credit 246,000
Ending RIP inventory per physical count, including
P2,100 conversion cost estimate 12,100
Compute the amount to be backflushed from RIP to Cost of Goods Sold:
a. P246,000 c. P247,000
b. 246,200 d. 245,000

82
. Using the same information in No. 2, compute the amount of Cost of Goods Sold after all
transactions and adjustments were made:
a. P246,000 c. P247,000
b. 246,200 d. 245,000

Questions 1 & 2 are based on the following: Dayag 2013


83
. Katherine, owner of KCO Supply Company in Cebu, which manufactures chopsticks for
restaurants, has recently decided to implement a JIT costing system. Transactions for August
are as follows:

ABC & Backflush – MCQ – Problems Page 29


COST ACCOUNTING

(a) Raw materials were purchased at the cost of P950,000.


(b) All materials purchased were requisitioned for production.
(c) Direct labor costs of P2,500,000 were incurred.
(d) Actual factory overhead costs amounted to P6,000,000.
(e) Applied conversion costs totaled P8,100,000. This included P2,500,000 of direct
labor.
(f) All units were completed.

Compute the amount to be backflushed from RIP to Finished Goods:


a. P2,100,000 c. P 950,000
b. 2,550,000 d. 3,100,000

84
. Using the same information in No. 1, compute the amount of Finished goods after all
transactions have been completed
a. P8,500,000 c. P10,600,000
b. 9,050,000 d. 9,650,000

Questions 1 thru 3 are based on the following: Dayag 2013


85
. G. Bello, general manager of a highly automated coffee production plant in Laguna, llocos
Sur, has provided the following information for transactions that occurred during October. The
production plant uses a JIT costing system.

a. Raw materials costing P300,000 were purchased.


b. All materials costing P300,000 were requisitioned for production.
c. Direct labor costs of P200,000 were incurred.
d. Actual factory overhead costs amounted to P995,000.
e. Applied conversion costs totaled P1,300,000. This includes the direct labor cost.
f. All units are completed and immediately sold.

Determined the October 31 balance in the Cost of Goods Sold account. No adjustment has
been made for overapplied or underapplied conversion cost.
a. P1,300,000 c. P1,600,000
b. 1,495,000 d. 1,195,000

86
. Using the same information in No. 1, what was the overapplied or underapplied conversion
costs for the month?

ABC & Backflush – MCQ – Problems Page 30


ABC & Backflush

a. P305,000 overapplied c. P105,000 overapplied


b. 195,000 underapplied d. 105,000 underapplied

87
. Using the same information in Nos. 1 and 2, what is the Cost of Goods Sold after all
transactions-adjustments have been completed?
a. P1,304,000 c. P1,600,000
b. 1,495,000 d. 1,195,000

Questions 1 & 2 are based on the following: Dayag 2013


88
. Ube Jewelry Factory manufacturer a variety of costume jewelry. The owner Rita Conrad had
recently decided to implement a JIT Costing System.
Transactions during September were as follows:

a. Raw materials totaling P45,000 were purchased.


b. All materials purchased were requisitioned for production.
c. Direct labor costs of P11,000 were incurred.
d. Indirect labor costs amounted to P120,000.
e. Utilities costs totaled P15,000.
f. Other actual factory overhead costs amounted to P85,000.
g. Applied conversion costs totaled P221,000. This includes the direct labor costs.
h. All units were completed.

Determine the September 30 balance in the cost of goods sold amount. No adjustment
has been made for overapplied or underapplied converison cost:
a. P266,000 c. P221,000
b. 276,000 d. 220,000

89
. Using the same information in No. 1, what was the overapplied or underapplied conversion
costs for the month?
a. P16,000 overapplied c. P10,000 underapplied
b. 16,000 underapplied d. (10,000) overapplied

Questions 1 & 2 are based on the following: Dayag 2013


90
. Kara Manufacturing uses backflush costing to account for an electronic meter it makes.
During August 2008, the firm produced 16,000 meters, of which it sold 15,800. The standard
cost for each meter is:

ABC & Backflush – MCQ – Problems Page 31


COST ACCOUNTING

Direct materials P20


Conversion costs 44
P64
Assume that the firm had no inventory on August 1. The following events took place in
August:
1. Purchased P320,000 of direct materials.
2. Incurred P708,000 of conversion costs.
3. Applied P704,000 of conversion costs to Raw and In Process Inventory (RIP).
4. Finished 16,000 meters.
5. Sold 15,800 meters for P100 each.
Compute the amount to be backflushed from RIP to Finished goods:
a. P320,000 c. P1,024,000
b. 704,000 d. 1,028,000

91
. Using the same information in No. 1, determine the August 3 (ending balance) of Finished
goods account:
a. P -0- c. P12,962
b. 12,850 d. 12,800

Questions 1 & 2 are based on the following: Punzalan 2014


The Backflushers Manufacturing Corp. uses a Raw and In Process inventory account and
expenses all conversion costs to the cost of goods sold account. At the end each month, all
inventories are counted, their conversion cost components are estimated and inventory account
balances are adjusted accordingly. Raw material cost is backflushed from RIP to finished goods.
The following information is for the month of May:

Raw and In Process inventory account, May 1, including


P500 of conversion cost P 5,000
Finished goods inventory account, May 1, including P3,250
of conversion cost 11,250
Raw materials received during May (50% down, balance in
four installments) 100,000
Raw and In Process inventory account, May 31, including
P650 of estimated conversion cost 5,250
Finished goods inventory account, May 31, including P2,500
conversion cost estimate 7,500

ABC & Backflush – MCQ – Problems Page 32


ABC & Backflush
92
. What is the amount to be backflushed from RIP to cost of goods sold?
a. 107,900 c. 112,900
b. 102,900 d. 86,900
93
. What is the balance of cost of goods sold after adjustment?
a. 102,000 c. 103,500
b. 102,300 d. 103,800

Questions 1 & 2 are based on the following: Punzalan 2014


The Cleaners Manufacturing Corp. uses a Raw and In Process (RIP) inventory account and
expensed all conversion costs to the cost of goods sold account. At the end of each month, all
inventories are counted, their conversion cost components are estimated and inventory account
balances are adjusted accordingly. Raw material cost is backflushed from RIP to finished goods.
The following information is for May:
Raw and In Process, May 1 38,700
Raw and In Process, May 31 41,900
Raw materials purchased 680,000
Conversion costs incurred 4,800
Conversion costs allocated 5,300
Finished goods, May 1 12,000
Finished goods, May 31 10,000
94
. What is the calculated material cost to be backflushed from RIP to cost of goods sold?
a. 718,700 c. 678,800
b. 688,800 d. 676,800

95
. What is the amount of conversion costs to be backflushed to cost of goods sold?
a. 10,100 c. 4,800
b. 5,300 d. 500

Use the following data in answering Numbers 1 and 2. Guerrero 2013


Mike Tuazon general manager of a highly automated coffee production plant in Bulacan has
provided the following information for transactions that occurred during October. The production
plant uses a JIT costing system.

a. Raw materials costing P300,000 were purchased.


b. All materials costing P300,000 were requisitioned for production.
c. Direct labor costs of P200,000 were incurred.
d. Actual factory overhead costs amounted to P995,000.

ABC & Backflush – MCQ – Problems Page 33


COST ACCOUNTING

e. Conversion costs allocated totaled P1,300,000. This includes the direct labor cost.
f. All units are completed and immediately sold.

96
. What is the over-allocated or under-allocated conversion costs for the month?
a. P305,000 over-allocated
b. P195,000 under-allocated
c. P105,000 over-allocated
d. P105,000 under-allocated

97
. Assuming no adjustment has been made for over-allocated or under-allocated conversion
cost, what is the balance of the cost of goods sold account on October 31?
a. P1,300,000 c. P1,600,000
b. P1,495,000 d. P1,195,000

JUST-IN-TIME
Throughput Time
98
. A company has 500 units production per day and 2,000 units are in process at any time. What
is the throughput time?
a. 1/2day c. Two days
b. 1/4day d. Four days Punzalan 2014

Detection of Defective Units


99
. In a company where Step #1 in a production line processes each unit and sends it to await
Step #2, and 500 units are waiting between Steps #1 and #2, how many defective units might
Step #1 produce before the problem is detected in Step #2?
a. 500 units c. 250 units
b. Unlimited number d. 1,000 units Punzalan 2014

Savings in Annual Carrying Costs


100
. Boulevard Co. has an average peso loss per defective unit of P25, a planned reduction in
work in process levels of 50%, and an average number of units in work-in-process per station
of 100. Assume that the total number of instances in which some work station goes out ol
control limits and produces defects is expected to be 500 annually and thai in half those
instances the out-of-control condition is not discovered immediately and enters 10% of the
units produced. What is the expected annual saving in cost of defective units?
a. 1,250 c. 6,250

ABC & Backflush – MCQ – Problems Page 34


ABC & Backflush

b. 31,250 d. 125 Punzalan 2014

101
. Atlas Co. has 10 work stations where work in process is held, 100 average units in work in
process per station, an average cost of a unit in work in process of P75, and annual inventory
carrying cost of 20%. If Atlas plans a 50% reduction in work in process levels, what would be
the expected annual savings in carrying costs?
a. 37,500 c. 30,000
b. 15,000 d. 7,500 Punzalan 2014

102
. A company has an annual carrying cost percentage of 30% and average work in process of
P300,000. The management plans to use Just-In-Time production system to double the
velocity of work in process without changing total annual output. What would be the savings
in annual carrying costs?
a. 90,000 c. 150,000
b. 45,000 d. 180,000 Punzalan 2014

ABC & Backflush – MCQ – Problems Page 35


ANSWER KEY
1.A 26.C 51.A 76.C 101.D
2.B 27.D 52.B 77.A 102.B
3.A 28.C 53.A 78.D
4.A 29.C 54.B 79.A
5.B 30.D 55.D 80.B
6.C 31.A 56.B 81.C
7.D 32.A 57.A 82.B
8.A 33.B 58.A 83.C
9.C 34.D 59.C 84.B
10.D 35.D 60.C 85.C
11.D 36.D 61.B 86.C
12.A 37.C 62.D 87.B
13.D 38.A 63.C 88.A
14.A 39.B 64.D 89.C
15.B 40.A 65.B 90.C
16.A 41.A 66.A 91.D
17.A 42.A 67.B 92.C
18.B 43.B 68.D 93.C
19.C 44.A 69.B 94.C
20.D 45.D 70.B 95.B
21.C 46.D 71.D 96.C
22.B 47.C 72.C 97.C
23.D 48.A 73.C 98.D
24.D 49.B 74.A 99.A
25.B 50.C 75.A 100.B

Answer Key Page 36


SOLUTIONS

Answer Key Page 37


1 .Suggested answer (a) Activity-based costing
An activity-based costing system identifies the causal relationship between incurrence of costs and activities,
determines the driver for each activity, and applies costs to products or services on the basis of resources (drivers)
consumed

2 .Suggested answer (b)

Multiple rates are appropriate when a process differs substantially among departments or when products do not go
through all departments or all processes. The trend in cost accounting is toward activity-based costing, which divides
production into numerous activities and attempts to identify the driver/s most relevant to each. A driver is a factor that
causes changes in a cost. Thus, activity-based costing emphasizes cost assignment based on trading rather than
allocation. The result is a more accurate assignment of costs

3 .Suggested answer (a)


A cost driver is a measure of activity, such as direct labor hours, machine hours, beds occupied, computer time used,
flight hours, etc., that is causal factor in the incurrence of cost to an entity. It is a basis used to assign costs to cost
objects.

4 .Suggested answer (a) resource driver

The name resources driver is new, but the idea is not. For example, most traditional accounting system allocate some
plant-wide costs to departments based on each department's allocation bases like the number of employees or square
footage occupied. If an ABC system allocates the cost of a resource to several activities based on the square footage or
number of employees devoted to each activity, then square footage and the number of employees are called resource
drivers.

5 .Suggested answer (b) activity driver

An activity driver is a base used to allocate the cost of an activity to products, customers, or other final cost objects. The
word final refers to the last step in cost allocation. The nature and variety of activity drivers is what distinguishes ABC
from traditional costing.

6 .Suggested answer (c) the use of information obtained from ABC to make improvements in the firm

Activity-based management is a detailed analysis of activities and expenses created by those activities used as a basis
for controlling and improving efficiency or the use of information obtained from activity-based costing to make
improvement in a firm.

7 .Suggested answer (d) Increase, Increase

In an activity-based cost system, cost assignment is more precise than in traditional cost system, because activities
rather than functions or departments are defined as cost objects. This structure permits assignment to more cost pools
and the identification of a driver specifically related to each.. A driver is a factor that causes a change in a cost. Thus an
activity based cost system uses more cost pools and cost assignment bases than a traditional system

8 .Suggested answer (d) Increase, Increase

In an activity-based cost system, cost assignment is more precise than in traditional cost system, because activities
rather than functions or departments are defined as cost objects. This structure permits assignment to more cost pools
and the identification of a driver specifically related to each.. A driver is a factor that causes a change in a cost. Thus an
activity based cost system uses more cost pools and cost assignment bases than a traditional system

9 .Suggested answer (c) No, Yes


An ABC system determines activities associated with the incurrence of costs and then accumulates a cost pools for
each activity. It identifies the driver/s used to trace those costs to such cost object as products or services. A driver is a
factor that causes a change in a cost. Activities that do not add value are identified arid eliminated to the extent
possible. However, all drivers cannot be eliminated.

10 .Suggested answer (d) Raw materials storage activity


ABC assign costs more precisely than traditional cost system because cost pools are accumulated for activities rather
than functions or departments. This structure permits assignment to more cost pools and the identification of the
driver/s used to reassign costs to cost objects. Analysis by activity also provides for better cost control, because of
identification of nonvalue activities. For example, raw materials storage may be greatly reduced or eliminated in a JIT
system without affecting customer value.

11 .Suggested answer (d) high quality and balanced work loads

Just in time is a philosophy centered on the reduction of costs through elimination of inventory. Elimination of
inventories eliminates storage and carrying costs, however, it also eliminates the cushion against production errors and
imbalances that inventories provide. As a result, high quality and balanced work loads are required in a JIT system to
avoid costly shutdowns and customer ill will. Because of the need for quality and balanced production, JIT has come to
be closely identified with efforts to eliminate waste in all its forms, and thus it is an important part of many total quality
management (TQM) efforts.

12 .Suggested answer (a) backflush production

The most visible aspect of JIT is the effort to reduce inventories of work in process and raw materials. Most writing on
JIT concentrate on this one aspect, which is called stockless production, lean production, or zero inventory production.
While backflush costing is an abbreviated approach to accounting for the flow of manufacturing costs, which is
applicable to mature JIT system in which velocity is too high that traditional accounting is impractical.

13 .Suggested answer (d) Cost of goods sold

In backflush costing, direct labor and overhead costs are expensed to the cost of goods sold account. The estimated
conversion cost components of the RIP and finished goods inventory account balances are adjusted at the end of each
month, with the offsetting entry representing a correction of cost of goods sold. If conversion cost increased during the
month, cost of goods sold is credited; however, when conversion cost decreased, cost of goods sold is debited.

14 .(a)
A cost allocation base is the common denominator for systematically correlating indirect costs and a cost object. The
cost driver of the indirect costs is ordinarily the allocation base. In a homogeneous cost pool, all costs should have the
same or a similar cause and-effect relationship with the cost allocation base.

15 .(b)

Multiple rates are appropriate when a process differs substantially among departments or when products do not go
through all departments or all processes. The trend in cost accounting is toward activity-based costing, which divides
production into numerous activities and identifies the cost driver(s) most relevant to each. The result is a more accurate
assignment of costs.
Answer (a) is incorrect because one rate may be cost beneficial when a single product proceeds through homogeneous
processes. Answer (c) is incorrect because individual cost drivers for all relationships must be known to use multiple
application rates. Answer
(d) is incorrect because, if one rate is related to several cost drivers, it can be used as a surrogate for multiple rates.

16 .(a)

An activity-based costing (ABC) system identifies the causal relationship between the incurrence of costs and activities,
determines the cost driver for each activity, and applies costs to products on the basis of resources (drivers) consumed.

Answer (b) is incorrect because target costing determines the maximum allowable cost of a product before the product
is designed or produced: it deducts an acceptable profit margin from a forecasted selling price. Answer (c) is incorrect
because cycle time is the period from the time a customer places an order to the time that product is delivered. Answer
(d) is incorrect because variable costing expenses fixed factory overhead as incurred.

17 .(a)

A cost driver is "a measure of activity, such as direct labor hours, machine hours, beds occupied, computer time used,
flight hours, miles driven, or contracts, that is a causal factor in the incurrence of cost to an entity." It is a basis used to
assign costs to cost objects.
Answer (b) is incorrect because cost drivers are not controls. Answer (c) is incorrect because cost drivers are measures
of activities that cause costs. Answer (d) is incorrect because, although cost drivers may be used to assign costs, they
are not necessarily mechanical. For example, a cost driver for pension benefits is employee salaries.

18 .(b)

A%C determines the activities associated with the incurrence of costs and then accumulates a cost pool for each
activity using the appropriate activity base (cost driver). However, given one product, all costs are assigned to the one
product; the particular method used to allocate the costs does not matter.

Answer (a) is incorrect because marketing and distribution costs should be allocated to specific products. Answer (c) is
incorrect because, by definition, a cost driver is a factor that affects (drives) the incurrence of a cost. Answer (d) is
incorrect because product-cost-cross-subsidization occurs when products are miscosted. The more accurate cost
allocation provided by ABC minimizes the subsidy received by undercosted products at the expense of overcosted
products.

19 .(C)

Given the automation of factories and the corresponding emphasis on ABC, plantwide application rates are being used
less often because better information can be provided at reasonable cost. ABC results in a more accurate application of
indirect costs because it provides more refined data. Instead of a single cost pool for a process, a department, or an
entire plant, an indirect cost pool is established for each identified activity. The related cost driver, the factor that
changes the cost of the activity, is also identified.
Answer (a) is incorrect because one effect of automation is a decrease in direct labor usage. Thus, some companies no
longer track direct labor costs closely. They treat direct labor as a factory overhead cost. Answer (b) is incorrect
because throughout is the rate of production over a stated time. It clearly drives (influences) costs. Answer (d) is
incorrect because multiple cost pools permit a better allocation of indirect costs to cost objects.

20 .(d)

In an ABC system, cost allocation is more precise than in traditional systems because activities rather than functions or
departments are defined as cost objects. This structure permits allocation to more cost pools and the identification of a
cost driver specifically related to each. A cost driver is a factor that causes a change in the cost pool for a particular
activity. Thus, an ABC system uses more cost pools and allocation bases than a traditional system.
Answer (a), (b), and (c) are incorrect because the numbers of cost pools and allocation bases increase.

21 .(c)

An ABC system determines activities associated with the incurrence of costs and then accumulates a cost pool for each
activity. It then identifies the cost driver for each activity. A cost driver is a factor that causes a change in the cost pool
for a particular cost object. Activities that do not add customer value are identified and eliminated to the extent possible.
A clear understanding of what causes a cost (the cost driver) helps eliminate the non-value-adding activities. However,
all cost drivers cannot be eliminated.
Answer (a),(b), and (d) are incorrect because non value-adding activities are
unnecessary and therefore are eliminated, but not all cost drivers are eliminated.

22 .(b)

A%C identifies the causal relationship between costs and activities, determines the drivers of the activities, establishes
cost pools related to individual drivers, develops costing rates, and applies cost to products on the basis of resources
consumed (drivers). The relationship between the number of components in a finished product and materials handling
costs is direct and causal.

Answer (a) is incorrect because the number of production runs per year does not provide individual unit costs. Answer
(c)\ is incorrect because the time to produce one unit is traditionally used to allocate overhead to units produced in a
labor-intensive process. Answer (d) is incorrect because overhead applied to each completed unit is the result of, not a
basis for, allocation.

23 .(d)

ABC allocates costs more precisely than traditional systems because cost pools are accumulated for activities rather
than functions or departments. This structure permits allocation to more cost pools and the identification of a cost driver
specifically related to each. Analysis by activity also provides for better cost control because of identification of
nonvalue-adding activities. For example, raw materials storage may be greatly reduced or eliminated in a JIT system
without affecting customer value.

Answers (a), (b), and (c) are incorrect because design engineering activity, heat treatment activity, and drill press activity add
value to products in the production process.

24 .(d)

25 .(b)

Ratios based on set-up:


Model A:100- (100+ 150 + 250)20%
Model B:150 + (100 + 150 + 250) 30%
Model C: 250 - (100 + 150 + 250) 50%

26 (c)

Ratios based on units produced:


Model A: 2,000 * (2,000 + 6,000 + 12,000)10%
Model B:' 6,000 + (2,000 + 6,000 + 12,000) 30%
Model C: 12,000 + (2,000 + 6,000 + 12,000) 60%

27 .(d)

Overhead charge using labor hours: P60 x 200P12,000


Overhead charge using machine hours: P12,000 - P4,000 P 8,000
Ratio of overhead costs to machine steps
P8,000/2,000 P4/step

28 .(c)

Cost Assignment:
Materials Handling: (P50,000 * 100,000) x 10,000P5,000
Painting: (P200,000 + 50,000) x 3,000 12,000
Assembly: (P120,000 + 4,000) x 300 9,000
P26,000

29 .(c)

Professional Salaries: P900,000 4- 30,000P 30 per hour


Building Costs: P450,000 + 15,000 P 30 per sq. ft.
Risk Management: P320,000 * 1,000 P320 perpatient (c)

30 .(d)

Surgery: (6,000 x P30) + (1,200 x P30) + (200 x P320)P280,000


Housing: (20,000 x P30) + (12,000 x P30) + (500 x P320) P1,120,000
Outpatient Case: (4,000 x P30) + (1,800 x P30) + (300 x P320) P270,000

31 .Overhead Rates:
Per Machine Hour: P900,000/3,600 machine hours 250

Per Setup: P525,000/300 setups P1,750


Cost of Job RST:
Parts and materials P56,000
Applied overhead:
Machine hours (70 x P250) P17,500
Setup (4 xP 1,750) 7,000 24,500
Total cost of Job RST P 80,500
32 .(a)

Calls: P300,000 * 150,000 callsP2 per call


Purchase Orders: PI 00,000 -f-10,000 purchase orders P10per
Reports: P73,500 -=- 7,000 receiving reports P10.50perR.R.

Cost Assignment:
125 calls x P2 per call P250.00
60 purchase orders x P10 per p.o 600.00
15 receiving report x P10.50 per R.R 157.50
Total costs P1,007.50

33 .(b)

Amount of purchasing department costsP1,007.50


Divided by: No. of units 200
P5.04/unit
34 .Overhead rates:
Material handling (P720.000 +6,000,000 parts) P0.12
Setupcosts(P315,000/750setups) 420
Machining costs (P540,000 * 30,000 hours) 18.00
Quality control (P225,000 + 500batches) 450
Overhead costs:

Material handling (20,000 units x 5 parts) x P0.12P 12,000


Setup costs activity (20,000 units / 5,000 x 2 setups) x P420 3,360
Machining activity (20,000 units / 5,000 x 80 hrs.) x P185,760
Quality control activity (20,000 units / 5,000) P450 1,800
Total P 22,920
Overhead cost per unit (P22,920/20,000 units) P1.15
Direct material cost per unit 4.40
Direct labor cost per unit 0.75
Total unit cost P6.30

35 . High school chair:


Direct materials P600,000
Direct labor (7,500 x P20) 150,000
Overhead:
Material handling (100,000 x P0.25) P 25,000
Cutting (100,000 xP2.50) 250,000
Assembly (7,500 x P25) 187,500 462,500
Total cost P1,212,500
Units produced -5,000
Unit cost P 242.50

College chair:
Direct materials P25,000
Direct labor (500 x P20) 10,000
Overhead:
Material handling (3,500 x P0.25) 875
Cutting (3,500 xP2.50) 8,750
Assembly (500 x P25) 12,500 22,125
Total cost P57,125
Units produced -100
Unit cost P571.25

36 .(d)

Per Direct Labor Hour: P200,000 / 16,000 DL hoursP12.50


Per Machine Hour: P300,000 / 4,000 machine hours P75.00

37 .(c)

Job # 345:
Direct MaterialsP2,000
Direct Labor (30 x PI0) 300
Applied Overhead: [(30 x PI2.50) + (10 x P75)] 1,125
P3,425

38 .(a)

Per Machine Hour: P900,000/3,600 machine hoursP250


Per Setup: P525,000/300 setups P1,750

39 .(b)

Job# 103:
Parts and MaterialsP56,000
Applied Overhead [(70 x P250) + (4 x PI ,750)] 24,500
P80,500
40 (a)

A&C allocates overhead costs more precisely than traditional methods. It identifies the activities associated with the
incurrence of costs, determines the cost driver for each activity, and allocates cost accordingly. Thus me cost per setup
is P100 (P20,000 + 200), per inspection P20 (PI30,000 +6,500), per material move P10 (P80.000 +8,000), and per
engineering hour P50 (P50,000 + 1,000). The overhead allocated to Job 101 is thereforeP 1,300 {(1 setup x P100) +
(20 inspections x P20] + (30 material moves x P10) + (10 enginering hours x P50)].

Answers (b) is incorect because P2,000 equals the overhead allocation for Job 103. Answer (c) is incorrect because
P5.000 equals the allocation of overhead using direct materials cost as a base. Answer (d) is incorrect because P5.600
equals 2% (2,000 DL cost + P 100,000 budgeted annual DL cost) of budgeted overhead.

41 .(a)

The overhead costs for the activities are PI00 per setup, P20 per inspection, P10. per material move, and P50 per
engineering hour. Thus, overhead allocated to Job 102 is P3.000 [(2 setups x P100) + (10 inspections x P20) + (10
material moves x P10) + (50 engineering hours x P50)}. The production cost of Job 102 is P17.000 (P12,000 DM +
P2.000 DL + P3.000 OH), and the cost per unit is P340 (P17,000 + 50).
Answer (b) is incorrect because P392 assumes overhead is allocated based on direct labor cost. Answer (c) is incorrect
because P440 assumes an overhead allocation of P8.000. Answer (d) is incorrect because P520 assumes overhead is
allocated based on direct materials cost.

42 .(a)

The costs per job for the activities are P100 per setup, P20 per inspection, P10 per material move, and P50 per
engineering hour. Overhead allocated to Job 103 is P2.000 [(4 setups x P100) + (30 inspections x P20j + (50 material
moves x P10) + (10 engineering hours x P50)]. Hence, the production cost of Job 103 is P14,000 (P8.000 DM + P4.000
DL + P2,000 OH), the cost per unit is P70:(P14,000 + 200), and the price is P98 (140% x P70j.
Answer (b) is incorrect because PI00 is the unit cost if overhead is allocated based on direct materials cost. Answer (c)
is incorrect because PI 16 is the unit cost if overhead is allocated based on direct labor cost. Answer (d) is incorrect
because P140 assumes cost is P100.

43 .(b)

Allocation of Costs using an overall rate:


Total Activity Center Costs (P7,500 + P125,000 + P9,750
+ P30.000)P172,250
Divided by: No. of Machine hours (12,000 + 13,000) 25,000
Overall rate P6.89
For Low-Unit: 12,000 x P6.89 P 82,680
ForMed-Unit: 13,000xP6.89 P 89,570

44 . Suggested answer (a) P200.00 P80.00 P125.00


Setups Machining Inspections
Overhead rate:
P100,000/500 P 200.00
P400,000/5,000 P80.00
P50.000/400 P125.00
If in traditional costing system, a single unit level basis of allocation is used to allocate overhead costs to products; ABC
recognizes that to have accurate and meaningful cost data, more than one basis of allocating activity costs to product is
needed. In selecting the basis, ABC seeks to identify the cost drivers that measure the activities performed on the
products. Since this case, there were two products produced, the measurement of activities should be the total for the
two products

45 . Suggested answer (d)?272,500 P277,500


Side A Side B
Setups (200x100) P20,000
(200x400) P80,000
Machining (80 x 3,000) 240,000
(80 x 2,000) 160,000
Inspections (125 x 100) 12,500
(125 x 300) 37,500
Total P 272,500 P 277,500
After determining the overhead rate, to compute the allocated overhead cost, overhead rate will be multiplied by the
number of activities for each product.

46 .Suggested answer (d)P20.00 P25.00


Side A Side B
Overhead cost per unit
(P272,500/l 3,625 units) P20.00
(P277.500/11,100 units) P25.00
47 .Suggested answer (c) 4.00 per step

Overhead using direct labor hours (60 x 200) 12,000


Overhead using machine hours (12,000 - 4,000) 8,000
Ratio of overhead to machine steps (8,000/2,000) 4.00/step
Given the ratio of overhead to direct labor hours of P60, which is higher by P4.000 than machine steps as the cost
driver, the resulting difference of P8.000 is the overhead cost using machine steps. Therefore, the ratio of overhead to
machine steps is P4.00 per step.

48 .Traditional costing f(P60,000 -f 25,000) x 2 direct labor hours)] P4.80


ABC(P1,000*100) P 10.00

49 .(b)

Using material moves under ABC costing:


P100,000 4- (10 + 40) = P2,000 per material moves
Model X:P2,000 per mat. moves x 10 mat. moves = P20,000
P20,000 -r 100 units produced = P200 (b)
Material Y: P2,000 per mat. moves x 40 mat. moves = P80,000
P80,000 -r 200 units produced = P400 (b)

50 .(c)

Using direct labor hours under traditional costing:


P100,000 * [(100 units x 1 hr./unit) + (200 units x 3 hrs./unit)]
P100,000 -r 700 hours = P143 per DL hour
Model X: P143 per DL hr. x 1 hr. per unit = 143
Model Y: PI 43 per DL hr. x 3 hrs. per unit = P429

51.(a)

The P50.000 of costs is allocated over 10,000 hours 1(25 x 200 hrs.) + (25 x 200 hrs.}]. Thus, the overhead cost per
hour is P5 (P50,000 + 10,000 hrs.), and the per unit overhead cost of wall mirrors is PI,000 (PS x 200 direct labor
hours).

Answer (b) is incorrect because P500 is the allocation based on number of material moves. Answer (c) is incorrect
because P2.000 assumes that all the overhead is allocated to the wall mirrors. Answer (d) is incorrect because P5.000
assumes overhead of P250,000.

52 .(b)

A8C allocates overhead costs on the basis of some causal relationship between the incurrence of cost and activities.
Because the moves for wall mirrors constitute 25% (5 -r 20) of total moves, the mirrors should absorb 25% of the total
materials handling costs, or P12.500 {25% x P50.000). The remaining P37.500 is allocated to specialty windows. The
cost per unit of wall mirrors is P500 (P12.500 + 25).
Answer (a) is incorrect because PI,000 uses direct labor as the allocation basis. Answer
(c) is incorrect because P4,500 is the allocation per unit of specialty windows. Answer
(d) js incorrect because P2.500 is not based on the number of material moves.

53 . The P50,000 of costs is allocated over 10,000 hours [(P25 x 200 hrs.) + (25 x 200 hrs.)]. Thus, the overhead cost per
hour is P5 (P50,000 - 10,000 hrs.), and the per unit overhead cost of wall mirrors is P1,000 (P5 x 200 direct labor
hours).

Answer (b) is incorrect because P500 is the allocation based on number of material moves. Answer (c) is incorrect
because P2,000 assumes that all the overhead is allocated to the wall mirrors. Answer (d) is incorrect because P5,000
assumes overhead of P250,000.

54 .ABC allocates overhead costs on the basis of some causal relationship between the incurrence of cost and activities.
Because the moves for wall mirrors constitute 25% (5 t 20) of total moves, the mirrors should absorb 25% of the total
materials handling costs, or P12,500 (25% x P50,000). The remaining P37,500 is allocated to specialty windows. The
cost per unit of wall mirrors is P500 (P 12,500 *25).
Answer (a) is incorrect because P1,000 uses direct labor as the allocation basis. Answer (c) is incorrect because
P4,500 is the allocation per unit of specialty windows. Answer (d) is incorrect because P2,500 is not based on the
number of material moves.

55 .(d)

Traditional Costing (DL hours):


= (P45,000 + P70,000 + P90,000) * (4,000 hrs. + 2,000 hr. + 4,000 hr.)
= P20.5 per DL hr.
Model S: P20.5 per DL hr. x 4,000 hrs. = P82,000 + 2,000 units =P41
Model M: P20.5 per DL hr. x 2,000 hr. = P41,000/6,000 units =P6.83

56 .(b)

ABC Costing - Model S:


Setups: P45,000 4- (100 + 150 + 25) = P90 x 100 =P 9,000
Shipping: P70,000 * (200 + 225 + 275) = P100 x 200 = 20,000
Eng'g.: P90,000 + (15 + 10 + 5) = P3,000 x 15 = 45,000
Total allocated costs P74,000
Divided by: No. of units produced 2,000
Cost per unit P37

57 .(a)

ABC Costing - Model M: (refer to No. 37)


Setups: P90x 150P13,500
Shipping: PI00x225 22,500
Eng'.: P300x 10 30,000
Total allocated cost P66,000
Divided by: No. of units produced 6,000

P11
58 .(a)

ABC Costing - Model L: (refer to No. 37)


Setups: P90 x 250P22,500
Shipping: PI00x275 27,500
Eng'.: P3,000 x 5 15,000
Total allocated cost P65,000
Divided by: No. of units produced 12,000
P5.42
59 . Suggested answer (c) 5,000

Traditional costing (160,000/40,000 x 400) 16,000


Activity-based costing:
Machine hours (120,000/20,000 x 500) 3,000
Set-ups (40,000/100x20) 8,000 11,000
Difference (cross subsidy) 5,000
In traditional costing system, a single pool of costs (overhead) is accumulated for a given organization unit, and these
costs are then assigned using an allocative rather than a tracing procedure. The effect is an averaging of costs that may
result in significant inaccuracy when products or service units do not use similar amounts of resources.

Activity-based costing attempts to improve costing by assigning cost to activities rather than to an organizational unit.
Accordingly, activity-based costing requires identification of the activities that consume resources and that are subject
to demands by ultimate cost objects.

60 .(c)

Indoor Outdoor
Annual revenue P300,000 P600,000
Less: Costs:
Materials P 40,000 P 60,000
Labor 50,000 75,000
Overhead:
Indoor: P500,000/P 125,000
x P50,000 200,000
Outdoor:P500,000/P 125,000
xP75,000 300,000
Total Costs P290.000 P435,000
Income (Loss) P 10,000
61
.(b)

IndoorOutdoor
Annual revenue P300,000 P600,000
Less: Costs:
Materials P 40,000 P 60,000
Labor 50,000 75,000
Overhead:
Maintenance:
Indoor: P200,000 / 100,000
x 80,000 160,000
OutdoorP200,000 / 100,000
x 20,000 40,000
Setups:
Indoor: P175,000/200 x 100 87,5000
Outdoor: P175,000/200 x 100 87,500
Administrative:
Indoor: PI25,000/ PI25,000
x P50.000 50,000
Outdoor:P125,000/ PI 25,000
xP75,000 75,000
Total Costs P387,500 P337,500
Income (Loss) P(.87,500) P26Z500

62 (d)

A&C identifies the causal relationship between the incurrence of cost and activities, determines the drivers of the
activities, establishes cost pools related to the drivers and activities, and assigns costs to ultimate cost objects on the
basis of the demands (resources or drivers consumed) placed on the activities by those cost objects. Hence, ABC
assigns overhead costs based on multiple allocation bases or cost drivers. Under the traditional, single-base system,
the amount allocated is P3.987.50 (P27.5Q0 x 14.5%). Under ABC, the amount allocated is P4.513 [(PI 1.50 x 12) + (P.
14 x 17,500) + (P77 x25)], or P525.50 more than under the traditional system.
Answer (A) is incorrect because the ABC assignment of P4.513 is at a rate of P180.52 for each of the 25 orders.
Answer (Bj is incorrect because ABC yields a higher allocation. Answer (C) is incorrect because the total is P4.513 on
the ABC basis.

63 .(c)

Given that manufacturing overhead is applied on the basis of machine hours, the overhead rate is P60 per hour
(P1,800,000 . 30,000) or P.96 per unit [(P60 x 80 machine hours per batch) + 5,000 units per batch]. Accordingly, the
unit full cost is P6.11 (P5.15 unit prime cost + P.96).
Answer (a) is incorrect because P5.39 assumes that 80 machine hours are required for the total production of 20,000
units. Answer (b) is incorrect because P5.44 is based on the machining overhead rate (P18). Answer (d) is incorrect
because P6.95 is based on the direct labor hour manufacturing overhead rate.

64 (d)

Materials handling cost per part is P.12 (P720.000 + 6,000,000), cost per setup is P420
(P315.000 + 750), machining cost per hour is P18 (P540.000 + 30,000), and quality cost per batch is P450 (P225.000 +
500). Hence, total manufacturing overhead applied is P22.920 [(5 parts per unit x 20,000 units x P.12) + (4 batches x 2
setups per batch x P420J + (4
batches x 80 machine hours per batch x PI8) + (4 batches x P450)]. The total unit cost is P6.296 [P5.15 prime cost +
(P22.920 + 20,000 units) overhead].
Answer (A) is incorrect because P6.00 assumes one setup per batch and 80 total machine hours. Answer (b) is
incorrect because P6.08 assumes that only 80 machine hours were used. Answer (c) is incorrect because P6.21
assumes one setup per botch.

65 .ABC:
Material handling (115x12) P1,380
Inspection (P1.40 x 17,500) 24,500
Product certification (P770 x 25) 19,250 P45,130
Traditional costing (P275,000 x 14.5%) 39,875
ABC higher than the traditional costing by P5,255

66 .Traditional costing system:


Direct materials (50 x P3,000) P150,000
Overhead (50 x 15) x P100 75,000
Total cost P225,000
Activity based costing system:
Direct materials P150,000
Overhead:
Material handling (50x50) x P8 20,000
Machining (50x12) xP68 40,800'
Assembly (50x 15) x P75 56,250
Inspection (50x14) xP104 72,800 P339,850
Higher P114,8 50
67 .Suggested answer (b) P2.40
Total overhead [P600 x (8 +2)] P6,000
Cost per machine hour [P6,000/(5 x 1,000)] P1.20

X Z
Traditional cost system
(P1.20x 2 machine hours) P 2.40
(P1.20 x 3 machine hours) P 3.60
Activity-based cost system
(P600x 8/1,000 units) 4.80
(P600 x 2/1,000 units) 1.20
Cross-subsidy per unit (Under) over cost (P 2.40) P 2.40
Peanut butter costing is an inaccurate averaging or spreading of costs over products or service units that use different
amounts of resources results in product cost cross subsidization. This term describes the condition in which the
miscasting of one product causes the miscasting of other product. Thus, the unit amount by which machine hour based
assignment over costs Z and at the same time under costs X (cross-subsidy) is P2.40.

68 .(d)

Beginning balance of RIP account (P31,000- PI ,400)P 29,600


Add: Raw materials received on credit 367,000
Total P396,600
Less: Ending balance of RIP inventory per physical count
(P33,000-PI,800) 31,200
Amount to be backflushed from RIP to Finished goods P365,400

69 .Suggested answer (b) P99,900

Raw and In Process inventory, May 1, net of


conversion (P5,000 - 500) P4,500
Add Materials received during June 100,000
Total 104,500
Less Raw and In Process inventory, June 30, net of
conversion (P5,250 - 650) 4,600
Amount to be backflushed from RIP to FG P 99,900
Since in backflush costing, materials and work in process are combined in a single account called RIP, for direct
materials used no journal entry is needed, and conversion costs are expensed to the cost of goods sold. Thus, the
conversion cost should be deducted from the amount of RIP to be backflushed to finished goods.
70 .Suggested answer (b) P202.500
Raw and In Process inventory, June 1 P10,000
Add Materials received during June 205,000
Total 215,000
Less Raw and In Process inventory, June 30 12,500
Amount to be backflushed P 202,500
The accounting for materials inventory, as well as work in process, can be altered by backflush costing. In a successful
Just-In-Time application where backflush costing is used, materials and work in process are combined in a single
account called Raw and In Process inventory. The backflush calculation uses end of period estimates of actual material
and conversion cost components of all unfinished work, including any unprocessed raw materials. Raw material cost is
backflushed from RIP to Finished Goods and from Finished Goods to Cost of Goods based on the monthly physical
counts.

71 .Materials in beginning balance of RIP account (P31,000-PI,400) P29,600


Add materials received on credit 367,000
Total 396,600
Less materials in ending balance of RIP account (P33,000 –P1,800) 31,200
Materials used to be backflushed from RIP to Finished Goods P365,400

72 .RIP account, beginning balance P38,700


Raw materials purchased 680,000
RIP account, ending balance (41,900)
Direct materials to be backflushed P676,8 00
Conversion costs allocated to the backflushed P5,300

73 .Materials in beginning balance of RIP account (P12,300-P1,300) P11,000


Add raw materials received on credit 246,000
Total 257,000
Less materials in ending balance of RIP accoutn (PI 2,100 - P2,100) 10,000
Materials used to be backflushed from RIP to Cost of Goods Sold P247,0 00

74 . Raw materials purchased (Debit to RIP) P880,000


Raw materials used (Credit to RIP) 850,000
Balance of RIP account P30,000

75 .Raw materials purchased - Dr. to RIP P5,300,000


Direct materials to be backflushed (192,000 units x P26)Cr. to RIP 4,992,000
Balance of RIP P308,000
Cost of Goods sold (192,000 units x P41) P7,872,00
76 . Raw materials purchased - Dr. to RIP P550,000
Direct materials to be backflushed - Cr. to RIP (21,000 x P25) 525,000
Balance of RIP account P25,000
Cost of completed units (21,000 units x P45) P945,000
Cost of goods sold (20,000 units x P45) 900,000
Balance of Finished Goods Inventory account P45,000

77 .Raw materials backflushed from RIP account P202,500


Materials in RIP inventory, June 30 12,500
Materials in RIP inventory, June 1 (10,000)
Raw materials purchased on credit during June P205,000

78 .Total conversion costs P825,000


Adjustments: Increase in conversion costs in RIP
(P4,500-P3,000) (1,500)
Decrease in conversion costs in FG
(P10,000-P8,750) 1,250
Conversion costs of units sold in June P824,750

79 .(a)

Actual Factory OverheadP225,000


Direct Labor Costs Incurred 77,000
Total actual conversion costs P302,000
Less: Applied Conversion Costs to production 300,000
March 31 Conversion Cost balance-debit / under-applied P2,000

80 .(b)

Amount to be backflushed from RIP to Finished GoodsP97,000


Applied conversion costs to production 300,000
March 31 finished goods balance, debit P397,000
*Note: There's no RIP beg. and ending balances.

81 .(c)

Beginning balance of RIP account (PI2,300- PI,300)P 11,000


Add: Raw materials received on credit 246,000
Total P257,000
Less: Ending balance of RIP inventory per physical count
(P12,100-P2,100) 10,000
Amount to be backflushed from RIP to Cost of goods sold. P247,00
82 .(b)

Cost of goods-soldP247,000
Less: Adjustment in Conversion Cost (P2,100 – P1,300). 800
P246,200

83 .(c)

Raw materials purchased were requisitioned for production .P950,000* (c)


*Note: There's no RIP beg. and ending balances.

84 .(b)
Amount to be backflushed from RIP to Finished GoodsP950,000
Applied conversion costs to production 8,100,000
P9,050,000

85 .(c)

Raw materials requistioned for production / amount to be


backflushed from RIP & CGSP300,000
Applied conversion costs to production 1,300,000
October 31 Cost of goods sold balance, debit P 1,600,000

86 .(c)

Actual factory overheadP995,000


Direct labor costs incurred 200,000
The Actual Conversion Costs P1,195,000
Less: Applied Conversion Costs to production 1,300,000
Overapplied conversion costs P105,000

87 . (b)

Cost of Goods Sold balance (refer to No. 8)P1,600,000


Less: Overapplied conversion costs (refer to No. 9) 105,000
P1,495,000

88 .(a)

Raw materials requisitioned for production / amount to be


fackflushed from RIP to CGSP45,000
Applied conversion cost to production 221,000
September 30 balance in the Cost of Goods Sold
balance, debit P266,000
89 .

Actual Factory Overhead:


Indirect laborP120,000
Utilities costs 15,000
Others 85,000
Direct labor costs incurred P220,000
11,000
Total Actual conversion costs P236,000
Less: Applied Conversion Costs to production 221,000
Underapplied Conversion Costs P10,000

90 .(c)

Beginning balance of RIP accountP0


Add:Raw materials purchased 320,000
Applied Conversion Costs* 704,000
Total P1,024,000
Less: Ending balance of RIP account 0
Amount to be backflush from RIP to Finished of goods P1,024,000
* It should be noted that the Applied Conversion costs transaction was entered into the RIP account as stated in the
problem, Normally, applied conversion costs goes directly to CGS account (in some cases, finished goods account).

91 .(d)

Amount backflush from RIP (No. 13)P1,024,000


Divided by: Units completed 16,000
Unit Cost P64
Multiplied by:'Finished goods ending inventory
(16,000 - 15,800) 200
P12,800

92 .Suggested answer (b) P102,900


Raw and In Process inventory, May 1, net of conversion
(P5,000-500) P4,500
Add Materials received during June 100,000
Total 104,500
Less Raw and In Process inventory, June 30, net of
conversion(P5,250-650) 4,600
Amount to be backflushed from RIP to FG P 99,900
Finished goods inventory, May 1, net of conversion
(P11,250-3,250) P 8,000
Add Materials backflushed to finished goods 99,900
Total 107,900
Less Finished goods inventory, May 31, net of
conversion (P7,500 - 2,500) 5,000
Amount to be backflushed from FG to CGS 102,900

As shown earlier, backflushing is from RIP to finished goods, because there is no finished goods inventory, but it should
be pointed out that in this case, backflushing is made directly to cost of goods sold. Further, if there is finished goods
inventory account, the amount to be shown as backflushed is up to cost of goods sold. Note in the above computations,
the amount added to the finished goods inventory is the amount backflushed from RIP to finished goods.

93 .Suggested answer (c) P103,500


Amount backflushed from FG to CGS P102,900
Add net decrease in conversion cost:
Increase (from RIP to FG)
(500-650) 150
Decrease (from FG to CGS)
(3,250-2,500) 750 600
Cost of goods sold balance P103,500

Again, the estimated conversion cost components of the RIP and finished goods inventory account balances are
adjusted at the end of each month, with the offsetting entry representing a correction of cost of goods sold. If
conversion cost increased during the month, cost of goods sold is credited; however, when conversion cost decreased,
cost of goods sold is debited.

94 .Suggested answer (c) 678,800


Raw and in process, May 1 38,700
Add raw materials purchased 680,000
Total 718,700
Less raw and in process, May 31 41,900
Amount to be backflushed from RIP to finished goods 676,800

Finished goods, May 1 12,000


Materials backflushed from raw and in process 676,800
Total 688,800
Less finished goods, May 31 10,000
Amount backflushed to cost of goods sold 678,800
Backflush costing determines some or all elements of cost of output only after production is completed. The cost of
completed work is subtracted from the balance of the work in process account, or equivalent combined account, in a
step called post-deduction or post-manufacturing deduction.

95 .Suggested answer (b) 5,300


The backflush calculation uses end of period estimates of actual materials and conversion cost components of all
unfinished work. -The cost estimates are made after a physical inventory count, which is usually done monthly or
weekly. Estimates of conversion cost amounts can be derived by first estimating the conversion cost of a finished unit,
and then assigning a part of that to the partly finished units on hand. Thus, the conversion costs to be backflushed to
cost of goods sold shall be the conversion allocated in the amount of P5,300.

96 . Actual factory overhead P995,000


Direct labor costs incurred 200,000
Total actual conversion costs 1,195,000
Conversion costs allocated to production 1,300,000
Over-allocated conversion costs P105,000

97 .Materials used to be backflushed from RIP to Cost of Goods Sold P300,000


Applied conversion costs to production 1,300,000
Cost of Goods Sold balance, October 31 P1,600,000

98 .Suggested answer (d) 4 days

Throughput time (2,000/500) 4 days In production, there is an important and direct relationship between the size of
work in process and the speed of production. Throughput time is inversely related to velocity and may be computed by
dividing units in process by units produced per day.

99 .Suggested answer (a) 500 units

At individual work stations in a production line, the impact of work in process reduction is simple, fewer units will be
waiting at, or moving to, each station; and this can have an important impact on production losses. In this case, where
500 units were waiting between Step HI and Step #2, the defective units Step #1 might produce before the problem is
discovered is the units Step #7 sends; possibly less, if the problem at Step #7 is discovered or if the defect enters only
some of the units produced. The worst outcome is 500 units, which is the size of work in process at that work station.
100 .Suggested answer (b) P31,250
Total average defective units
(100 x 10%) x ('/2 x 500) 2,500 units
Multiply by reduction in work in process level 50%
Total savings in defective units 1,250 units
Multiply by average peso loss per defective unit 25
Expected annual savings in cost of defective units P31,250

Again, in addition to the effect on inventory carrying costs, a more important result of a work in process reduction is its
effect on production losses. At individual work stations in a production line, the worst outcome on defective units is the
size of work in process in that work station; possibly less, if the problem at a work station is discovered in some other
way or if the defects enters only some of the units produced.

101 .Suggested answer (d)?7,500

Average work in process (P75 x 10 x 100) P75,000


Multiply by annual carrying cost 20% .
Total 15,000
Multiply by reduction in work in process 50%
Expected annual savings in carrying costs P7,500

Again, because work in process is a costly asset that must be financed and maintained just as any other assets, the
obvious benefit of a work in process reduction is that total investment is reduced, producing savings in inventory
carrying costs. In addition to the effect on inventory carrying costs, a more important result of a work in process
reduction is its effect on production losses.

102 .Suggested answer (b)P45,000


Savings in annual carrying costs
(30% x Vi x P300,000) P45,000
Velocity is the speed in which units or tasks are processed in a system. A strategic benefit of increased velocity is the
reduced time needed to fill production orders. And because work in process is a costly asset that must be financed and
maintained just as any other asset producing savings in the carrying cost of inventory. Where velocity is doubled, the
annual carrying cost percentage is reduced by one-half

You might also like